Vous êtes sur la page 1sur 6

Errata for A&J Study Manual for exam FM/2

2nd Edition
Errata for A&J Study Manual 2nd Edition SOA Exam FM/ CAS Exam 2

1. Page 55, Example 2.1.

In the solution, the effective is wrong and thus it affects the whole solution. The whole new
solution is like the followings:

Solution 2.1

SOA likes to trouble the candidates with this type of questions. Although the topic tested is
valuation of annuity, they usually won’t give you information that can be used directly.

Since we are given the nominal rate of interest payable semiannually instead of annual effective
rate, we need to convert it before attempting the questions:
i (2)  0.054
2
 0.054 
 1  i  1  
 2 
 i  5.4729%

(SOA really likes to play with interest conversion. Make sure you master it.)

(a) This is an annuity immediate, since it pays $40 only after 1 period of time. So,
 1  v5 
PV  40a5 5.4729%  40    40  4.27346   170.94
 0.054729 

#1
The price of the annuity is $170.94.

If you find it hard to determine if it is annuity immediate or annuity due, let the time diagram help
you.
40 40 40 40 40

0 1 2 3 4 5

The payment is at t=1. Hence it is annuity immediate with 5 payments.

(b) Since the first payment has not yet been made, and there are still 5 payments with the first
one coming almost immediately, it is an annuity due with 5 payments.

40 40 40 40 40

0 1 2 3 4 5
 
 1 v 5 
PV  40a5 5.4729%%  40    40  4.50734   180.29
  0.054729  
  1.054729  
 

The price of annuity right before the first payment is $180.29.

Copyright © 2010 A&J, 2nd Edition |


2
Errata for A&J Study Manual 2nd Edition SOA Exam FM/ CAS Exam 2

#2

(c) The first payment has been made. This means that there are 4 more payments to go, with
the first payment starts after 1 year. Hence, it is an annuity immediate with 4 payments:

40 40 40 40

0 1 2 3 4

 1  v4 
PV  40a4 5.4729%  40    40  3.50734   140.29
 0.054729 

The price right after the first payment is $140.29.

2. Page 80. Third Paragraph. There is a typo error in the first line. P+3Q after three years
should be changed to P+2Q after three years.

3. Page 129. Example 3.8. The new solution for the example is as the following:

Solution 3.8

As suggested, it is easier to solve it with a table:

Date 1 Jan 1 Apr 31 Aug 12 1 May 13 31 Dec


09 10 13
Balance before deposit $6700 $10,900 + $11,300 $12,300
or withdrawal X
Deposit $7500 $2500 - Y
Withdrawal - - X -
Balance after deposit or $7500 $9200 $10,900 $11300 +
withdrawal Y

We are given that the dollar-weighted rate is 23%. This means,

12300  7500  2500  Y  X


0.23 
 45   16   8 
7500  2500      X    Y
 60   60   60 
2300  Y  X
 0.23 
4 2
9375  X  Y
15 15

We are also given that the time-weighted rate is 25.6%. This means,

Copyright © 2010 A&J, 2nd Edition |


3
Errata for A&J Study Manual 2nd Edition SOA Exam FM/ CAS Exam 2

 6700   10900  X   11300   12300 


1.256      
 7500   9200   10900   11300  Y 
 10900  X 
 1.256  1.238177  
 11300  Y 
10900  X
 1.014395 
11300  Y
 11462.66  1.014395Y  10900  X
X  562.66
Y 
1.014395

Substitute the value of Y into the first equation,


 X  562.66 
2300    X
0.23   1.014395 
4 2  X  562.66 
9375  X   
15 15  1.014395 
2300   0.98581X  554.6754   X
 0.23 
9375  0.26667 X  0.13144  X  562.66 
2854.6754  0.01419 X
 0.23 
9301.04397  0.13523 X
 2139.24011  0.0311029 X  2854.6754  0.01419 X
 X  15795.75
X  562.66
Y   16126.27
1.014395

X+Y is -31,922.02. The negative sign means that, Alvin actually deposits $15,795.75 on 31 Aug
2012 and withdraws $16,126.27 on 1 May 2013.

4. Page 304. In the paragraph under the section of Covered Put. In the last third line, “Your
maximum loss occurs if the stock price goes to 0” should be changed to “Your maximum loss
for the put option occurs if the stock price goes to 0”.

5. Page 339. Third paragraph. “Consider a firm that sells a goods” should be changed to
“Consider a firm that sells goods”. The article “a” before goods is omitted.

6. Page 144. Example 4.1. The outstanding loan at the end of 2 nd year is $1,258.13 instead
of $1330.43.

7. Page 184. Example 6.6. “Andrew purchases a $1000 bond to yield 4% payable
semiannually” instead of 2%.

8. Page 62. The solution for Example 2.6 should be as followed:

Copyright © 2010 A&J, 2nd Edition |


4
Errata for A&J Study Manual 2nd Edition SOA Exam FM/ CAS Exam 2

Solution 2.6

First and foremost, we have to find the initial price of the bungalow. We can find it using the
interest rate and the monthly payments. We need to convert the given interest rate to monthly
effective rate, since the payments are made per month. Also, since his marriage is exactly 2 years
from now, there will be 24 months (payments):
PInitial  45000
s24 12 1.12 1
 45000
s24 0.94888%
 1217924.01

Hence, we obtain the new price:


PNew  1217924.011.7   2070470.82

The price rises only after the 7th payment is made. This means that the first 7 payments will be
contributed to the new price also. The contribution is:
s7 0.94888% 1.009488817   327185.49 1.174155  384,166.61
45000

This implies that the remaining 17 payments must accumulate to $2,070,470.82- $384,166.61=
$1,696,304.21.

Since Alvin can’t afford it, he needs the help of his fiancée. With her help, he needs to pay X such
that the sum of their accumulated deposit will give $1,700,545.89.

150,000 150,000 150,000


45000 X X X X X
6 7 8 …… 13 …… 19 …… 23 24

1, 696,304.21  Xs17 0.94888%  150000 1.009488817   150000 1.009488811   150000 1.00948885 


 1,196,597.11  Xs17 0.94888%
 X  64,583.38

Notice that the 7th payment is paid at the end of the 6 th period (or, the beginning of the 7 th period).
If you are not convinced and think that it should be at t=7, you can think in this way: The 1 st
payment starts at t=0, the 2nd payment is made at t=1, the 3 rd payment is made at t=2, and so forth.
You should get the 7th payment is made at t=6.

Hence, Alvin should deposit $64,538.38 starting from the 8th payment so that he can purchase
the bungalow on time.

9. In page 94, the 7th paragraph should be: 20 N 5 I/Y 1000 PMT CPT PV
10. In page 94, the 14th paragraph should be: You will see -2247.75 on the display.

Copyright © 2010 A&J, 2nd Edition |


5
Errata for A&J Study Manual 2nd Edition SOA Exam FM/ CAS Exam 2

11. In page 271, example 3.3, Janice purchased a put option, not a call option. Hence, the
calculation of strike price should be as followed:

Profit  max  0, K  ST   FV  P 
 0  max  0, K  40  FV  5.60 
 0  max  0, K  40  5.6e0.04 0.25
 5.65628  K  40
 K  45.65628

The strike price is $45.65.

Copyright © 2010 A&J, 2nd Edition |


6

Vous aimerez peut-être aussi